Diễn Đàn MathScopeDiễn Đàn MathScope
  Diễn Đàn MathScope
Ghi Danh Hỏi/Ðáp Community Lịch

Go Back   Diễn Đàn MathScope > Sơ Cấp > Tài Liệu > Đề Thi > Đề Thi HSG Cấp Tỉnh ở Việt Nam

News & Announcements

Ngoài một số quy định đã được nêu trong phần Quy định của Ghi Danh , mọi người tranh thủ bỏ ra 5 phút để đọc thêm một số Quy định sau để khỏi bị treo nick ở MathScope nhé !

* Nội quy MathScope.Org

* Một số quy định chung !

* Quy định về việc viết bài trong diễn đàn MathScope

* Nếu bạn muốn gia nhập đội ngũ BQT thì vui lòng tham gia tại đây

* Những câu hỏi thường gặp

* Về việc viết bài trong Box Đại học và Sau đại học


Trả lời Gởi Ðề Tài Mới
 
Ðiều Chỉnh Xếp Bài
Old 06-10-2012, 08:22 PM   #16
kainguyen
+Thành Viên+
 
kainguyen's Avatar
 
Tham gia ngày: Apr 2012
Bài gởi: 105
Thanks: 70
Thanked 65 Times in 43 Posts
Trích:
Nguyên văn bởi namdung View Post
Bạn có thể tính vài số hạng đầu tiên của dãy để thấy thực sự đây không phải là dãy tăng.

Vấn đề của bạn là không để ý đến điểm gián đoạn của hàm f(x).

Vâng đúng là sai ạ, em xin giải lại như sau

Đặt: $tana=2 $. Ta cm theo quy nạp được: $u_n=tanna $.

1. Ta có: $u_{2m}=tan2ma=\frac{2tanma}{1-tan^2ma}=\frac{2u_m}{1-u_m^2} $ (1)

Giả thiết phản chứng tồn tại n sao cho $u_n=0 $. Xét:

TH1: $n $ chẵn, $n=2m $ suy ra $u_m=0 $

TH2: $n $ lẻ, biểu diễn $n=2^k(2s+1) $

Sử dụng $u_n $ và (1) $k $ lần được $u_{2s+1}=0 $

Nếu $m $ chẵn, thực hiện tiếp đến khi $m $ lẻ.

Sau đó chứng minh được $u_s $ là số vô tỉ mà đề bài cho $u_n $ là số hữu tỉ suy ra vô lý (đpcm).

2. Gỉả sử tồn tại 2 số hạng của dãy cùng nhận 1 giá trị.

Ta có: $u_{m+n}=u_n $

$\Leftrightarrow tan(m+n)a=tanna $

$\Leftrightarrow \frac{sinma}{cos(m+n)acosa}=0 $

$\Leftrightarrow sina=0\Rightarrow tanma=0 $

$\Rightarrow u_m=0 $

Suy ra vô lý. Vậy ta có đpcm.
[RIGHT][I][B]Nguồn: MathScope.ORG[/B][/I][/RIGHT]
 
__________________
Kai Nguyen
kainguyen is offline   Trả Lời Với Trích Dẫn
The Following 5 Users Say Thank You to kainguyen For This Useful Post:
Akira Vinh HD (10-08-2015), BlackBerry® Bold™ (08-10-2012), DramonsCelliet (10-10-2012), gomis (08-10-2012), namdung (08-10-2012)
Old 06-10-2012, 11:58 PM   #17
DaiToan
+Thành Viên+
 
Tham gia ngày: Oct 2010
Đến từ: THPT Chuyên Vĩnh Phúc
Bài gởi: 280
Thanks: 29
Thanked 361 Times in 123 Posts
Xin đóng góp vài bài trong phân PTH-Đa thức. Các bài còn lại đa được giải trên diễn đàn...
[RIGHT][I][B]Nguồn: MathScope.ORG[/B][/I][/RIGHT]
 
File Kèm Theo
Kiểu File : doc VMO2013 PTH-DT (2).doc (234.5 KB, 548 lần tải)
DaiToan is offline   Trả Lời Với Trích Dẫn
The Following 10 Users Say Thank You to DaiToan For This Useful Post:
BlackBerry® Bold™ (08-10-2012), DramonsCelliet (10-10-2012), gomis (08-10-2012), hoangkute69 (07-10-2012), namdung (08-10-2012), Nick Trần (04-08-2013), thaibinh (07-10-2012), TrauBo (07-10-2012), vanthanh0601 (07-10-2012), viettam (07-10-2012)
Old 07-10-2012, 07:31 PM   #18
namdung
Administrator

 
Tham gia ngày: Feb 2009
Đến từ: Tp Hồ Chí Minh
Bài gởi: 1,343
Thanks: 209
Thanked 4,066 Times in 778 Posts
Gửi tin nhắn qua Yahoo chát tới namdung
Cảm ơi thầy Đại chuyên Vĩnh Phúc cũng đã góp sức.

Tiếp tục gửi mọi người các đề toán về Số học.
[RIGHT][I][B]Nguồn: MathScope.ORG[/B][/I][/RIGHT]
 
File Kèm Theo
Kiểu File : doc 2VMO2013_Sohoc1.doc (39.5 KB, 442 lần tải)
namdung is offline   Trả Lời Với Trích Dẫn
The Following 9 Users Say Thank You to namdung For This Useful Post:
BlackBerry® Bold™ (08-10-2012), DramonsCelliet (10-10-2012), gomis (08-10-2012), Nick Trần (04-08-2013), thanhorg (07-10-2012), thaygiaocht (08-10-2012), TrauBo (08-10-2012), vanthanh0601 (07-10-2012), viettam (07-10-2012)
Old 07-10-2012, 09:47 PM   #19
thaygiaocht
+Thành Viên+
 
thaygiaocht's Avatar
 
Tham gia ngày: Aug 2012
Đến từ: Chuyên Hà Tĩnh
Bài gởi: 165
Thanks: 793
Thanked 216 Times in 93 Posts
Trích:
Nguyên văn bởi namdung View Post
Tiếp tục gửi mọi người các đề toán về Số học.
Tổng hợp một số bài toán Số học.
[RIGHT][I][B]Nguồn: MathScope.ORG[/B][/I][/RIGHT]
 
File Kèm Theo
Kiểu File : doc VMO2013_Sohoc1.doc (243.5 KB, 470 lần tải)

thay đổi nội dung bởi: thaygiaocht, 07-10-2012 lúc 09:49 PM
thaygiaocht is offline   Trả Lời Với Trích Dẫn
The Following 9 Users Say Thank You to thaygiaocht For This Useful Post:
BlackBerry® Bold™ (08-10-2012), DramonsCelliet (10-10-2012), gomis (08-10-2012), hieu1411997 (08-10-2012), hongson_vip (07-10-2012), namdung (08-10-2012), Nick Trần (04-08-2013), TrauBo (08-10-2012), vanthanh0601 (08-10-2012)
Old 08-10-2012, 11:36 AM   #20
thaygiaocht
+Thành Viên+
 
thaygiaocht's Avatar
 
Tham gia ngày: Aug 2012
Đến từ: Chuyên Hà Tĩnh
Bài gởi: 165
Thanks: 793
Thanked 216 Times in 93 Posts
Tiếp tục với phần Hình học.
[RIGHT][I][B]Nguồn: MathScope.ORG[/B][/I][/RIGHT]
 
File Kèm Theo
Kiểu File : doc 2VMO2013_Hinhhoc1.doc (185.0 KB, 553 lần tải)
thaygiaocht is offline   Trả Lời Với Trích Dẫn
The Following 11 Users Say Thank You to thaygiaocht For This Useful Post:
Akira Vinh HD (10-08-2015), BlackBerry® Bold™ (08-10-2012), DramonsCelliet (10-10-2012), gomis (08-10-2012), hieu1411997 (08-10-2012), namdung (08-10-2012), Nick Trần (04-08-2013), tffloorz (08-10-2012), TrauBo (08-10-2012), vanthanh0601 (08-10-2012), viettam (11-10-2012)
Old 09-10-2012, 09:19 PM   #21
kien10a1
+Thành Viên+
 
kien10a1's Avatar
 
Tham gia ngày: Feb 2011
Đến từ: Vĩnh Yên- Vĩnh Phúc
Bài gởi: 371
Thanks: 43
Thanked 263 Times in 153 Posts
Gửi tin nhắn qua Yahoo chát tới kien10a1
Em xin lỗi về bài 9 phần ở post trước, nhận xét đó đúng nhưng không thể áp dụng.
Ta sẽ giải bài này bằng đơn biến.
Vì $\sqrt{a}+\sqrt{b}\leqslant 2\sqrt{a+b} $ nên tổng các căn bậc của các số trên bảng luôn không giảm.
Như vậy, nếu chỉ còn lại một số S thì $\sqrt{1}+\sqrt{2}+...+\sqrt{n}\leqslant \sqrt{S}\Rightarrow S\geqslant (\sqrt{1}+\sqrt{2}+...+\sqrt{n})^2=A_n $
Dễ dàng quy nạp được $A_n\geqslant\frac{4n^3}{9} $ nên suy ra đpcm.
[RIGHT][I][B]Nguồn: MathScope.ORG[/B][/I][/RIGHT]
 
__________________
Quay về với nơi bắt đầu
kien10a1 is offline   Trả Lời Với Trích Dẫn
The Following User Says Thank You to kien10a1 For This Useful Post:
DramonsCelliet (10-10-2012)
Old 09-10-2012, 11:07 PM   #22
liverpool29
+Thành Viên+
 
liverpool29's Avatar
 
Tham gia ngày: Dec 2010
Đến từ: hue
Bài gởi: 348
Thanks: 425
Thanked 560 Times in 237 Posts
Em xin đóng góp chút cho phần hình học:

Bài 13: Ta có: $PM=PN \Leftrightarrow AD.\sin BAC = CD .\sin ACB \Leftrightarrow \dfrac{AD}{CD}=\dfrac{\sin ACB}{\sin BAC} \Leftrightarrow \dfrac{AD}{CD}=\dfrac{AB}{CB}$.
Ta có điều phải chứng minh.

Bài 14:
Gọi $M,N,P$ lần lượt là giao điểm của $AG$ và $BC$; $BG$ và $AC$; $CG$ và $AB$.
Thuận: Ta chứng minh nếu $G$ là trọng tâm của tam giác $ABC$ thì $O$ là trọng tâm của tam giác $DEF$
Gọi $J$ là điểm thoả: $AJ \parallel BN; JM \parallel CP$.
Suy ra $AJ=BN; MJ=CP$.
Ta cũng suy ra được: $\bigtriangleup{AJM} \backsim \bigtriangleup{DEF}$. (1)
Gọi $K$ là giao điểm của $JM, AB$.
Ta có: $KM=\dfrac{CP}{2}=\dfrac{JM}{2}$.Suy ra $K$ là trung điểm của $JM$.
Mặt khác, do (1) và do $\widehat{OEF}=\widehat{KAM}$ nên ra suy ra $OE$ là trung tuyến của tam giác $DEF$. (*)
Tương tự với $DO,FO$.
Ta có đccm.
Đảo: Ta chứng minh nếu $O$ là trọng tâm của tam giác $DEF$ thì $G$ là trọng tâm của tam giác $ABC$.
Vẽ thêm và lập luận hoàn toàn tương tự, ta có đccm.

Các bài còn lại hầu hết đã có lời giải trên diễn đàn.
Phía dưới là dẫn chứng:

[RIGHT][I][B]Nguồn: MathScope.ORG[/B][/I][/RIGHT]
 
__________________
LIFE HAS SENT TO US A MIRACLE, IT'S GEOMETRY

"Don't try your best. Do your best."

thay đổi nội dung bởi: liverpool29, 09-10-2012 lúc 11:20 PM
liverpool29 is offline   Trả Lời Với Trích Dẫn
The Following 4 Users Say Thank You to liverpool29 For This Useful Post:
DramonsCelliet (10-10-2012), pqhoai (09-10-2012), q785412369 (10-10-2012), TNP (09-10-2012)
Old 10-10-2012, 10:17 AM   #23
namdung
Administrator

 
Tham gia ngày: Feb 2009
Đến từ: Tp Hồ Chí Minh
Bài gởi: 1,343
Thanks: 209
Thanked 4,066 Times in 778 Posts
Gửi tin nhắn qua Yahoo chát tới namdung
Cảm ơn thầy giáo chuyên Hà Tĩnh cũng đã tham gia chủ đề, cảm ơn các bạn tiếp tục hưởng ứng chủ đề.

Tôi gửi tiếp một chủ đề yêu thích của nhiều bạn: Bất đẳng thức và cực trị.
[RIGHT][I][B]Nguồn: MathScope.ORG[/B][/I][/RIGHT]
 
File Kèm Theo
Kiểu File : doc 2VMO2013_BDT1.doc (37.0 KB, 498 lần tải)
namdung is offline   Trả Lời Với Trích Dẫn
The Following 5 Users Say Thank You to namdung For This Useful Post:
DramonsCelliet (10-10-2012), hosyhaiql (04-12-2012), L Ha (07-10-2013), Nick Trần (15-09-2013), thaygiaocht (10-10-2012)
Old 10-10-2012, 02:41 PM   #24
HashiramaSenju
+Thành Viên+
 
Tham gia ngày: Sep 2012
Bài gởi: 25
Thanks: 5
Thanked 21 Times in 10 Posts
Trích:
Nguyên văn bởi namdung View Post
Cảm ơn thầy giáo chuyên Hà Tĩnh cũng đã tham gia chủ đề, cảm ơn các bạn tiếp tục hưởng ứng chủ đề.

Tôi gửi tiếp một chủ đề yêu thích của nhiều bạn: Bất đẳng thức và cực trị.
Đang học thì thấy file của thầy Dũng gửi lên. Làm trước một bài mà mình thấy thích nhất.

Bài 1 (Nghệ An). Cho $a,b,c$ dương và $abc=1,$ tìm GTNN của $$P=a^2b+b^2c+c^2a+\frac{1}{\sqrt[6]{a^3+b^3+c^3}}.$$

Thay $(a,b,c)$ bởi $\left (\sqrt[3]{\dfrac{a}{b}},\sqrt[3]{\dfrac{b}{c}},\sqrt[3]{\dfrac{c}{a}} \right ),$ ta có thể viết $P$ lại dưới dạng thuần nhất như sau $$P=\frac{a+b+c}{\sqrt[3]{abc}}+\sqrt[6]{\frac{abc}{ab^2+bc^2+ca^2}}.$$ Mấu chốt của bài toán này chính là "căn bậc 6" khó chịu kia, nếu khử được căn hoặc là tăng bậc nó lên thì công việc chứng minh của ta sẽ nhẹ hơn rất nhiều. Vậy ta sẽ làm tăng bậc của nó lên. Sử dụng đánh giá hiển nhiên $$\frac{abc}{ab^2+bc^2+ca^2} \ge 3\left (\frac{abc}{ab^2+bc^2+ca^2} \right )^2,$$ ta được $$P=\frac{a+b+c}{\sqrt[3]{abc}}+\sqrt[6]{3}\cdot\sqrt[3]{\frac{abc}{ab^2+bc^2+ca^2}}.$$ Ba đại lượng $abc,\; ab^2+bc^2+ca^2$ và $a+b+c$ là ta liên tưởng đến bất đẳng thức quen thuộc $$(a+b+c)^3\ge \frac{27}{4}(ab^2+bc^2+ca^2+abc).$$ Sử dụng đánh giá này, ta có được $$P\ge \sqrt[3]{\frac{27}{4}\left (\frac{ab^2+bc^2+ca^2}{abc}+1 \right )}+\sqrt[6]{3}\cdot \sqrt[3]{\frac{abc}{ab^2+bc^2+ca^2}}.$$ Đến đấy chỉ cần đặt $t=\dfrac{ab^2+bc^2+ca^2}{abc} \ge 3,$ thì ta có thể đưa bài toán tìm giá trị nhỏ nhất của biểu thức ba biến về một biến $$P\ge \sqrt[3]{\frac{27(t+1)}{4}}+\frac{\sqrt[6]{3}}{\sqrt[3]{t}}.$$ Khử căn của biểu thức trong ngoặc và Holder giúp ta thực hiện công việc này $$t+1\ge \frac{(\sqrt[3]{9t}+1)^3}{16}.$$ Từ đó thu được $$P\ge \frac{3}{4}(\sqrt[3]{9t}+1)+\frac{\sqrt[6]{3}}{\sqrt[3]{t}}.$$ Công việc còn lại chỉ là cân bằng hệ số trong bất đẳng thức AM-GM. Ta có thể viết $$\frac{3}{4}(\sqrt[3]{9t}+1)+\frac{\sqrt[6]{3}}{\sqrt[3]{t}}=\frac{3}{4}+\left ( \frac{3\sqrt[3]{9}}{4}-\sqrt[6]{\frac{1}{27}} \right )\sqrt[3]{t}+\sqrt[6]{3}\left ( \sqrt[3]{\frac{t}{9}}+\frac{1}{\sqrt[3]{t}} \right ).$$ Từ đó theo bất đẳng thức AM-GM, ta được
$$P\ge \frac{3}{4}+\left ( \frac{3\sqrt[3]{9}}{4}-\sqrt[6]{\frac{1}{27}} \right )\sqrt[3]{3}+2\cdot \sqrt[6]{3}\cdot \sqrt{\sqrt[3]{\frac{t}{9}}\cdot \frac{1}{\sqrt[3]{t}} }=3+\frac{1}{\sqrt[6]{3}}.$$ Đẳng thức xảy ra khi và chỉ khi $t=3,$ tức là $a=b=c. \Box$
[RIGHT][I][B]Nguồn: MathScope.ORG[/B][/I][/RIGHT]
 
HashiramaSenju is offline   Trả Lời Với Trích Dẫn
The Following 3 Users Say Thank You to HashiramaSenju For This Useful Post:
babysama (17-09-2014), hoanghaithanh (07-01-2013), hosyhaiql (04-12-2012)
Old 10-10-2012, 06:09 PM   #25
q785412369
+Thành Viên+
 
Tham gia ngày: Nov 2010
Bài gởi: 97
Thanks: 144
Thanked 42 Times in 27 Posts
Các bài toán hình học đã giải khá nhiều trên diễn đàn rồi, em chỉ xin đóng góp chút ít cho bài 14 bằng bổ đề sau :
Cho tam giác ABC có các trung tuyến AD, BE, CF và tam giác A'B'C' có các trung tuyến A'D', B'E', C'F'. Chứng minh rằng AD, BE, CF lần lượt vuông góc với B'C', C'A', A'B' khi và chỉ khi A'D', B'E', C'F' lần lượt vuông góc với BC, CA, AB.

------------------------------
Trích:
Nguyên văn bởi namdung View Post
Cảm ơn thầy giáo chuyên Hà Tĩnh cũng đã tham gia chủ đề, cảm ơn các bạn tiếp tục hưởng ứng chủ đề.

Tôi gửi tiếp một chủ đề yêu thích của nhiều bạn: Bất đẳng thức và cực trị.
Thầy Nam Dũng xem giùm em đề bài 3, hình như đề bị thiếu ạ ?
[RIGHT][I][B]Nguồn: MathScope.ORG[/B][/I][/RIGHT]
 

thay đổi nội dung bởi: q785412369, 10-10-2012 lúc 06:45 PM Lý do: Tự động gộp bài
q785412369 is offline   Trả Lời Với Trích Dẫn
Old 12-10-2012, 12:12 AM   #26
DaiToan
+Thành Viên+
 
Tham gia ngày: Oct 2010
Đến từ: THPT Chuyên Vĩnh Phúc
Bài gởi: 280
Thanks: 29
Thanked 361 Times in 123 Posts
Đây là lời giải các bài BĐT...
[RIGHT][I][B]Nguồn: MathScope.ORG[/B][/I][/RIGHT]
 
File Kèm Theo
Kiểu File : doc 2VMO2013_BDT2.doc (246.5 KB, 615 lần tải)
DaiToan is offline   Trả Lời Với Trích Dẫn
The Following 8 Users Say Thank You to DaiToan For This Useful Post:
Ams (23-03-2013), hosyhaiql (04-12-2012), L Ha (07-10-2013), namdung (12-10-2012), nguyentatthu (12-10-2012), Nick Trần (15-09-2013), thaibinh (12-10-2012), thaygiaocht (12-10-2012)
Old 15-10-2012, 09:08 AM   #27
namdung
Administrator

 
Tham gia ngày: Feb 2009
Đến từ: Tp Hồ Chí Minh
Bài gởi: 1,343
Thanks: 209
Thanked 4,066 Times in 778 Posts
Gửi tin nhắn qua Yahoo chát tới namdung
Cảm ơn thầy Đại đã giải và bình luận rất kỹ.

Tôi bổ sung thêm một số lời giải và bình luận cho hoàn chỉnh phần BĐT.
[RIGHT][I][B]Nguồn: MathScope.ORG[/B][/I][/RIGHT]
 
File Kèm Theo
Kiểu File : doc 2VMO2013_BDT2.doc (338.0 KB, 556 lần tải)
namdung is offline   Trả Lời Với Trích Dẫn
The Following 4 Users Say Thank You to namdung For This Useful Post:
DaiToan (15-10-2012), L Ha (07-10-2013), Nick Trần (15-09-2013), thaygiaocht (15-10-2012)
Old 15-10-2012, 10:32 AM   #28
Galois_vn
+Thành Viên+
 
Tham gia ngày: Nov 2007
Đến từ: Konoha
Bài gởi: 899
Thanks: 372
Thanked 362 Times in 269 Posts
Trích:
Nguyên văn bởi TrauBo View Post
Cảm ơn mọi người. Vậy là ta tạm xong phần PT - HPT. Sẽ chờ cập nhật các đề khác.
Cho TrauBo hỏi bài 1 có ai có cách giải tự nhiên hơn không ạ?
Đặt mỗi cái căn $A=\sqrt{5x-1}, B=\sqrt{5-2x} $

Ta có thể qui về phương trình dạng:
$aA^3+bA^2B+cAB^2=dA^2+eB^2+fAB, 2A^2+5B^2=23 $

Dùng BĐT được chắc mấy hệ số trên sẽ đặc biệt.
[RIGHT][I][B]Nguồn: MathScope.ORG[/B][/I][/RIGHT]
 
Galois_vn is offline   Trả Lời Với Trích Dẫn
The Following User Says Thank You to Galois_vn For This Useful Post:
TrauBo (14-07-2013)
Old 17-10-2012, 09:12 PM   #29
namdung
Administrator

 
Tham gia ngày: Feb 2009
Đến từ: Tp Hồ Chí Minh
Bài gởi: 1,343
Thanks: 209
Thanked 4,066 Times in 778 Posts
Gửi tin nhắn qua Yahoo chát tới namdung
Tiếp tục cập nhật một số bài Tổ hợp
[RIGHT][I][B]Nguồn: MathScope.ORG[/B][/I][/RIGHT]
 
File Kèm Theo
Kiểu File : doc 2VMO2012_Tohop2.doc (34.0 KB, 360 lần tải)
namdung is offline   Trả Lời Với Trích Dẫn
The Following 4 Users Say Thank You to namdung For This Useful Post:
hoangnam94 (18-10-2012), L Ha (07-10-2013), nguyentatthu (18-10-2012), thaygiaocht (18-10-2012)
Old 21-10-2012, 09:14 AM   #30
quangbynh
+Thành Viên+
 
Tham gia ngày: Jan 2008
Bài gởi: 147
Thanks: 36
Thanked 209 Times in 50 Posts
Th Miniflorrr

Trích:
Nguyên văn bởi thaygiaocht View Post
Tiếp tục với phần Hình học.
Bổ sung HH từ bài 15 đến 25
[Only registered and activated users can see links. ]
[RIGHT][I][B]Nguồn: MathScope.ORG[/B][/I][/RIGHT]
 
File Kèm Theo
Kiểu File : doc HH mathscope VMO2012-2013.doc (264.0 KB, 475 lần tải)
quangbynh is offline   Trả Lời Với Trích Dẫn
The Following 6 Users Say Thank You to quangbynh For This Useful Post:
L Ha (07-10-2013), lovetohop (27-10-2012), namdung (21-10-2012), Nick Trần (15-09-2013), Raul Chavez (22-10-2012), thaygiaocht (21-10-2012)
Trả lời Gởi Ðề Tài Mới

Bookmarks


Quuyền Hạn Của Bạn
You may not post new threads
You may not post replies
You may not post attachments
You may not edit your posts

BB code is Mở
Smilies đang Mở
[IMG] đang Mở
HTML đang Tắt

Chuyển đến


Múi giờ GMT. Hiện tại là 06:43 AM.


Powered by: vBulletin Copyright ©2000-2024, Jelsoft Enterprises Ltd.
Inactive Reminders By mathscope.org
[page compression: 124.36 k/141.93 k (12.38%)]